Q5

User avatar
 
ManhattanPrepLSAT1
Thanks Received: 1909
Atticus Finch
Atticus Finch
 
Posts: 2851
Joined: October 07th, 2009
 
This post thanked 1 time.
 
 

Q5

by ManhattanPrepLSAT1 Mon Jul 26, 2010 2:08 am

Here's a setup for this game as well as a walk through for questions 3 and 5.

Let me know if you still need further help with this one!
Attachments
PT40, S2, G1_AtlasLSAT.pdf
(88.88 KiB) Downloaded 819 times
 
emarxnj
Thanks Received: 0
Vinny Gambini
Vinny Gambini
 
Posts: 6
Joined: February 18th, 2011
 
 
 

Re: Q5

by emarxnj Fri Mar 25, 2011 4:44 pm

How did you decide how to set up your hypotheticals? My first one included A and C (and E by coincidence). Then I made one for choices B and D, but when it didn't work, I had trouble figuring out which of the two answers included in the hypo were the cause of it being invalid. This could have been remedied with just doing them seperately I suppose, but that brings me back to the question of knowing how to group multiple answers into one hypo, and then how to discern the problematic condition.




If thats confusing, let me know and I'll try again :)
User avatar
 
noah
Thanks Received: 1192
Atticus Finch
Atticus Finch
 
Posts: 1541
Joined: February 11th, 2009
 
This post thanked 1 time.
 
 

Re: Q5

by noah Fri Mar 25, 2011 5:39 pm

I think the way that Matt's walk through "went down" is that the hypothetical created for A turned out to eliminate other answer choices. I wouldn't recommend trying to build more than one answer into a hypothetical, for the reason you've mentioned. However, if you make a hypothetical, and you see some elements that could switch places, indicate that so that your hypothetical covers more than one situation.

I solved that question a bit differently:

If Z is first, hmm, what can I infer? So, I know that L must be before O, so L can't be last, so M can't be third. Who is left? (this is one of the most important questions to ask when solving a lot of conditional questions). K-M-T or T-M-K is left. Any restrictions on that? Yeah, M can't go 3rd (as that would force L last). So, M can go in slots 4 or 5. Regardless of whether it's T or K following that M, since neither can go in 5, we know we have one of them in slot 6 (so put in a K/T) there.

Seems like a lot of options after that, so from there, I start evaluating answer choices:

(A) I don't see a problem. Defer judgment, look for an obvious answer.
(B) I don't see a problem. Defer judgment, look for an obvious answer.

Oh, yeah, K and T are the same element (no rules are different about them, so A and B are the same answer.

(C) this leaves plenty of room for O and M K/T, defer, though I strongly suspect it's fine
(D) this looks suspicious - slow down. With L back there, where would we fit M, O, K/T?

Pull the trigger.

With this sort of question, going into hypotheticals is appropriate if you're not feeling comfortable manipulating the elements in your head (after you've laid out the basic situation and inferences suggested by the new condition). I think a "lazier" approach works faster here, but it's only possible if you're feeling comfortable with the rules and with that approach in general.

I hope that helps.
 
zainrizvi
Thanks Received: 16
Atticus Finch
Atticus Finch
 
Posts: 171
Joined: July 19th, 2011
 
 
trophy
First Responder
 

Re: Q5

by zainrizvi Fri Apr 26, 2013 3:54 pm

Noah, thanks for the explanation. I just want to get a better grip on your reasoning....when you say "who is left", do you mean the elements themselves, or just the rules that are applicable to the elements. Either way, what made you not take into account the rule that states T and K cannot be 5. Your method obviously seems the most efficient, but I think if I were to balance the t/k cannot go 5 in my head along with the rest of the info, it would get a bit confusing.

I'm interested in any thoughts you might have on how to accelerate through these last few time consuming questions in a game.

Thanks!
User avatar
 
noah
Thanks Received: 1192
Atticus Finch
Atticus Finch
 
Posts: 1541
Joined: February 11th, 2009
 
 
 

Re: Q5

by noah Fri Apr 26, 2013 5:08 pm

zainrizvi Wrote:Noah, thanks for the explanation. I just want to get a better grip on your reasoning....when you say "who is left", do you mean the elements themselves, or just the rules that are applicable to the elements. Either way, what made you not take into account the rule that states T and K cannot be 5. Your method obviously seems the most efficient, but I think if I were to balance the t/k cannot go 5 in my head along with the rest of the info, it would get a bit confusing.

I'm interested in any thoughts you might have on how to accelerate through these last few time consuming questions in a game.

Thanks!

"Who is left" is referring to the elements, not the rules. We're pretty consistent with that usage in the book.

Good point about the TK restriction, I edited my explanation above to add that.

As for accelerating through the last few questions, there's no magic bullet. It's back to the basics: follow the inference chains, defer judgement, etc. Perhaps the most important thing is to move faster through easier questions and games to give yourself more time with harder ones.
 
mitrakhanom1
Thanks Received: 1
Elle Woods
Elle Woods
 
Posts: 63
Joined: May 14th, 2013
 
 
 

Re: Q5

by mitrakhanom1 Thu Sep 26, 2013 3:49 pm

i'm still confused on why M has to be moved to space 3. If Z is in space 1 and L is in space 4 why can't M be moved to space 5 and K/O be in spaces 2,3, or 6 and T maybe in space 2 or 3?
User avatar
 
noah
Thanks Received: 1192
Atticus Finch
Atticus Finch
 
Posts: 1541
Joined: February 11th, 2009
 
 
 

Re: Q5

by noah Thu Sep 26, 2013 4:46 pm

mitrakhanom1 Wrote:i'm still confused on why M has to be moved to space 3. If Z is in space 1 and L is in space 4 why can't M be moved to space 5 and K/O be in spaces 2,3, or 6 and T maybe in space 2 or 3?

In case someone is reading this other than Mitrakhanom, the question is why does adding L fourth illegally force M to be third.

I think you're forgetting about the Z1 --> L -O rule.

With L fourth, O must be 5th or 6th. With the other taken up by T/K. The other T/K has to go in slot 2, leaving only slot 3 for M.
 
mitrakhanom1
Thanks Received: 1
Elle Woods
Elle Woods
 
Posts: 63
Joined: May 14th, 2013
 
 
 

Re: Q5

by mitrakhanom1 Mon Sep 30, 2013 7:20 pm

i dont understand why M can't be on space 5. Couldn't a hypothetical be ZKTLMO?
User avatar
 
noah
Thanks Received: 1192
Atticus Finch
Atticus Finch
 
Posts: 1541
Joined: February 11th, 2009
 
 
 

Re: Q5

by noah Mon Sep 30, 2013 10:04 pm

mitrakhanom1 Wrote:i dont understand why M can't be on space 5. Couldn't a hypothetical be ZKTLMO?

Who says M can't go 5th? Matt's solution has M fifth in two valid hypotheticals.

BTW, your hypothetical is invalid since it must be either T - M - K or K - M - T.
 
621jolene
Thanks Received: 0
Vinny Gambini
Vinny Gambini
 
Posts: 1
Joined: November 12th, 2015
 
 
 

Re: Q5

by 621jolene Thu Nov 12, 2015 1:41 pm

I am confused on E, for some reason I can't open the file, but I don't see how E could work, M cannot be on third, and L has to before O, so it left M with spot 2, which it won't work. Can someone help me with this? Thank you. My study materials told me D is correct answer (L added fourth)